LSAT and Law School Admissions Forum

Get expert LSAT preparation and law school admissions advice from PowerScore Test Preparation.

 Administrator
PowerScore Staff
  • PowerScore Staff
  • Posts: 8927
  • Joined: Feb 02, 2011
|
#26044
Complete Question Explanation
(The complete setup for this game can be found here: lsat/viewtopic.php?f=107&t=7219)

The correct answer choice is (D)

This question asks us to identify the maximum number of designated subzones allowed in Z3. From our discussion of Numerical Distributions above, we know that the maximum number of designated subzones in each of Z2 and Z3 is five (I, I, I, R, R). Answer choice (D) is therefore correct.

Get the most out of your LSAT Prep Plus subscription.

Analyze and track your performance with our Testing and Analytics Package.